5
$\begingroup$

I've been working on a problem. I found a couple of related questions on the site, but I was having a little bit of trouble clarifying everything. The goal is to show that given $f\notin L^p(E)$, there exists a $g\in L^{p'}(E)$ such that $fg\notin L^1(E)$. Note this is the second part of Wheeden and Zygmund's Introduction to Real Analysis textbook.

To show that for $f\notin L^p(E)$, there exists a $g\in L^{p'}(E)$ such that $fg\notin L^1(E)$, consider the following. Without loss of generality, we can assume all functions are nonnegative. Now, suppose there is a sequence $\{g_k\}_{k=1}^\infty\subseteq L^{p'}(E)$ with $\Vert g_k\Vert_{p'}=1$ and $ \int_E fg_k>3^{k}. $ Set $ g=\sum_{k=1}^\infty 2^{-k}g_k $ and observe that, by Minkowski's inequality, $\Vert g\Vert_{p'}\leq 1$. Note that $ \int_E fg=\int_E f\sum_{k=1}^\infty 2^{-k}g_k>\int_E\sum_{k=1}^\infty \left(\frac{3}{2}\right)^{k}=\infty. $ Thus, $g\in L^{p'}(E)$ but $fg\notin L^1(E)$. Hence, we have reduced the problem to showing that such a sequence exists. First note that if $f=\infty$ on any set $A$ of positive measure, then we can simply take

$g=\frac{1}{|B|}\text{ when }x\in B$ where $B\subseteq A$ has positive, finite measure, and $g=0$ otherwise. Thus $g$ has the desired properties. This implies we can assume $f$ is finite a.e., and in particular, for any positive real number $c$, we can find a set $F$ with finite measure such that $\int_F f=c$. With this in mind, we can find a nested sequence of sets $\{E_k\}_{k=1}^\infty$, each with finite measure, such that $\bigcup_{k=1}^\infty E_k=E$ and $\int_{E_k}f>3^k$. Now, take $g_k=\frac{1}{|E_k|^{\frac{1}{p'}}} :\ x\in E_k$ By construction, $\Vert g_k\Vert_{p'}=1$, and it is easy to check that $\int fg_k\longrightarrow\infty$ as $k\to\infty$. Hence, we have the existence of the sequence, and we've demonstrated the existence of such functions $g$. Does this seem sufficient? The last part seems to be a little bit lacking, but I'm not quite sure why I feel it's unjustified. Thanks in advance!

  • 0
    [Same question from same book](http://math.stackexchange.com/q/90303/8271), although something different is being asked here.2012-12-02

1 Answers 1

0

The existence of the sequence of function $\left(f_k\right)$ can be done by proving the following:

For any positive $R$, there exists a function $g\in\mathbb L^{p'}$ such that $\left\lVert g\right\rVert_{p'}=1$ and $\int fg\gt R$.

Otherwise, there would exist $R_0\gt 0$ such that for any $g\in\mathbb L^{p'}$ with $\left\lVert g\right\rVert_{p'}=1$, the inequality $\int fg\leqslant R_0$ would hold. Apply it to $ g:=\left( \left\lVert f^{p-1}\mathbf 1\left\{f\lt n\right\}\right\rVert_{p'}\right)^{-1}f^{p-1}\mathbf 1\left\{f\lt n\right\}$ where $n$ is a fixed integer. Then defining $a_n:= \left\lVert f^{p}\mathbf 1\left\{f\lt n\right\}\right\rVert_{p}$, one gets $a_n^p\leqslant R_0\cdot a_n^{p-1}$ and the monotone convergence theorem would ensure that $f$ belong to $\mathbb L^p$, which is the contrary of what we assumed.